Đến nội dung

Hình ảnh

Bất đẳng thức chuẩn bị cho kì thi THPTQG 2015-2016


  • Please log in to reply
Chủ đề này có 389 trả lời

#201
daicahuyvn

daicahuyvn

    Hạ sĩ

  • Thành viên
  • 93 Bài viết

Cho a,b,c là các số thực dương tìm giá trị lớn nhất và giá trị nhỏ nhất của

 P=$\frac{2a^{2}}{(a+b)^{2}}+\frac{2b^{2}}{(b+c)^{2}}+\frac{2c^{2}}{(c+a)^{2}}+\frac{4a^{2}b^{2}}{(a+b)^{2}(b+c)^{2}}$

Đặt $x=\frac{b}{a},y=\frac{c}{b},z=\frac{a}{c}$, ta có x,y,z>0 và xyz=1

Tìm max:

$P=\frac{2}{(1+x)^2}+\frac{2}{(1+y)^2}+\frac{2}{(1+z)^2}+\frac{4}{(1+x)^2(1+y)^2}\le \frac{2}{1+x}+\frac{2}{1+y}+\frac{2}{1+z}+4$

$\frac{1}{1+x}+\frac{1}{1+y}+\frac{1}{1+z}\le 2 \Leftrightarrow \frac{xy+yz+zx+1}{(x+1)(y+1)(z+1)}\ge 0$

Do đó, $P\le 8$, $P\to 8$ khi $\frac{b}{a}\to 0; \frac{c}{b}\to 0$

Min

$P=\frac{2}{(1+x)^2}+\frac{2}{(1+y)^2}+\frac{2x^2y^2}{(1+xy)^2}+\frac{4}{(1+x)^2(1+y)^2}$

$\min P \approx 1,6848$ khi $x=y\approx 1,30368$ là nghiệm của pt bậc 7.



#202
daicahuyvn

daicahuyvn

    Hạ sĩ

  • Thành viên
  • 93 Bài viết

Cho các các số thực dương x, y. Tìm giá trị nhỏ nhất của: $P=\frac{1}{x^2}+\frac{1}{y^2}+\frac{1}{x^2+y^2}+5ln(\sqrt{xy})$

$\frac{1}{x^2}+\frac{1}{y^2}-\frac{2}{xy}+\frac{1}{x^2+y^2}-\frac{1}{2xy}=(x-y)^2(\frac{1}{x^2y^2}-\frac{1}{2xy(x^2+y^2)})\ge (x-y)^2(\frac{1}{x^2y^2}-\frac{1}{4x^2y^2})\ge0 \Leftrightarrow \frac{1}{x^2}+\frac{1}{y^2}+\frac{1}{x^2+y^2}\ge \frac{5}{2xy}$

$P\ge \frac{5}{2xy}+5\ln(\sqrt{xy})=\frac{5}{2xy}-\frac{5}{2}\ln (\frac{1}{xy}) $

Đặt $t=\frac{1}{xy}$. $P\ge \frac{5}{2}t-\frac{5}{2}\ln t\ge \frac{5}{2}$

Dấu = xảy ra khi x=y=1. GTNN của P là 5/2.



#203
daicahuyvn

daicahuyvn

    Hạ sĩ

  • Thành viên
  • 93 Bài viết

Cho a,b,c dương thỏa a+b+c = 2. Tìm GTLN của

P =$\frac{ab}{\sqrt{2c+ab}}+\frac{bc}{\sqrt{2a+bc}}+\frac{ca}{\sqrt{2b+ac}}$

2c+ab=c(a+b+c)+ab=(a+c)(b+c)

$P=\frac{ab}{\sqrt{(a+c)(b+c)}}+\frac{bc}{\sqrt{(a+b)(a+c)}}+\frac{ca}{\sqrt{(a+b)(b+c)}}\le \frac{1}{2}\sum (\frac{ab}{a+c}+\frac{ab}{b+c})=\frac{1}{2}\sum (\frac{ab}{a+c}+\frac{bc}{a+c})=\frac{a+b+c}{2}=1$

Đẳng thức xảy ra khi $a=b=c=\frac{2}{3}$ hoặc $a=b=1,c=0$

Vậy GTLN của P là 1.



#204
daicahuyvn

daicahuyvn

    Hạ sĩ

  • Thành viên
  • 93 Bài viết

Cho $x,y,z>0$ thỏa $5(x^2+y^2+z^2)=6(xy+xz+yz)$. Tìm GTNN của $$P=(x+y+z)(\frac{1}{x}+\frac{1}{y}+\frac{1}{z})$$

5(x+y+z)^2=16(xy+yz+zx).Chuẩn hoá x+y+z=4 thì có xy+yz+zx=5

$P=\frac{4(xy+yz+zx)}{xyz}=\frac{20}{xyz}$

Vậy chỉ cần tìm max của xyz.

$yz=5-x(y+z)=5-x(4-x)=x^2-4x+5\le \frac{(y+z)^2}{4}=\frac{(4-x)^2}{4}\Rightarrow \frac{2}{3}\le x \le 2$

$xyz=x(x^2-4x+5)=2+(x-1)^2(x-2)\le 2\Rightaroow P\ge 10$

Đẳng thức xảy ra khi x=2y=2z hoặc y=2z=2x hoặc z=2x=2y.

GTNN của P là 10.


Bài viết đã được chỉnh sửa nội dung bởi daicahuyvn: 25-05-2014 - 22:53


#205
NDP

NDP

    Hạ sĩ

  • Thành viên
  • 74 Bài viết

$a,b,c\geq 0 , ab+bc+ac=1$

Tìm min P=$\frac{1}{\sqrt{a^{2}+b^{2}}}+\frac{1}{\sqrt{b^{2}+c^{2}}}+\frac{1}{\sqrt{a^{2}+c^{2}}}$

Có tại đây này http://k2pi.net/showthread.php?t=17356


 $\sqrt{O}$ve math

 

:ukliam2:  :ukliam2:  :ukliam2:

 

 

:ukliam2: Learn from yesterday, live for today, hope for tomorrow and the important thing is not to stop questioning :like

                                       

                         my facebook: https://www.facebook.com/NDPA1K46
           my email: [email protected]
 
 
 

 


#206
25 minutes

25 minutes

    Thành viên nổi bật 2015

  • Hiệp sỹ
  • 2795 Bài viết

Cho các số thực dương a, b, c thỏa mãn abc = 1

CMR: $\frac{a}{b^2}+\frac{b}{c^2}+\frac{c}{a^2}+\frac{9}{2(a+b+c)}\geq \frac{9}{2}$

Áp dụng AM-GM ta có $\sum \frac{a}{b^2}\geqslant \frac{1}{a}+\frac{1}{b}+\frac{1}{c}=\frac{ab+bc+ca}{abc}=ab+bc+ca$

Và $(ab+bc+ca)^2\geqslant 3abc(a+b+c)=3(a+b+c)\Rightarrow \frac{9}{2(a+b+c)}\geqslant \frac{27}{2(ab+bc+ca)^2}$

Do đó ta chỉ cần chứng minh  

 $ab+bc+ca+\frac{27}{2(ab+bc+ca)^2}\geqslant \frac{9}{2}\Leftrightarrow \left [ 2(ab+bc+ca)+3 \right ](ab+bc+ca-3)^2\geqslant 0$

BDT trên luôn đúng với $abc=1$

Đẳng thức xảy ra khi $a=b=c=1$


Hãy theo đuổi đam mê, thành công sẽ theo đuổi bạn.



Thảo luận BĐT ôn thi Đại học tại đây


#207
megakill1994

megakill1994

    Binh nhất

  • Thành viên
  • 29 Bài viết

Cho a, b, c là 3 số thực thuộc đoạn $[\frac{1}{3};3]$. Tìm GTLN của biểu thức:

$P=\frac{a}{a+b}+\frac{b}{b+c}+\frac{c}{c+a}$

Không mất tính tổng quát. Giả sử $a\geq b\geq c\Rightarrow \left\{\begin{matrix} \frac{b}{a}\leq 1 & & \\ \frac{c}{b}\leq 1 & & \end{matrix}\right.$

Ta có: $P=\frac{1}{1+\frac{b}{a}}+\frac{1}{1+\frac{c}{b}}+\frac{1}{1+\frac{a}{c}}\leq \frac{2}{1+\sqrt{\frac{c}{a}}}+\frac{1}{1+\frac{a}{c}}$

Đặt: $t=\sqrt{\frac{c}{a}}\Rightarrow \frac{1}{3}\leq t\leq 1$

Khi đó: $P\leqslant \frac{t^3+3t^2+2}{t^3+t^2+t+1}=f(t)$

$f'(t)=\frac{-2(x-1)^2(x^2+x+1)}{(x^3+x^2+x+1)^2}\leqslant 0 \forall t[\frac{1}{3},1]$

$\Rightarrow P_{max}= f(\frac{1}{3})=\frac{8}{5}$

Dấu "=" xảy ra khi: $\frac{b}{a}=\frac{c}{b}=\frac{1}{3}\Rightarrow a=3;b=1;c=\frac{1}{3}$

Hoặc $\frac{b}{a}=\frac{a}{c}=\frac{1}{3}\Rightarrow a=1;b=\frac{1}{3};c=3$

Hoặc $\frac{a}{c}=\frac{c}{b}=\frac{1}{3}\Rightarrow a=\frac{1}{3};b=3;c=1$


Bài viết đã được chỉnh sửa nội dung bởi megakill1994: 03-06-2014 - 18:23


#208
daicahuyvn

daicahuyvn

    Hạ sĩ

  • Thành viên
  • 93 Bài viết

Cho $x,y,z$ là các số thực thoả mãn $xy+yz+zx=xyz$.Tìm giá trị nhỏ nhất của biểu thức

$A=\frac{x\sqrt{(y+z)^3}}{2yz+zx+xy}+\frac{y\sqrt{(z+x)^3}}{2zx+xy+yz}+\frac{z\sqrt{(x+y)^3}}{2xy+yz+zx}$

Đặt $a=\frac{1}{x},b=\frac{1}{y},c=\frac{1}{z}$ thì $a+b+c=1$

$\frac{x\sqrt{(y+z)^3}}{2yz+zx+xy}=\frac{\sqrt{\frac{(b+c)^3}{bc}}}{2a+b+c}\ge \frac{2\sqrt{b+c}}{2a+b+c}=\frac{2\sqrt{1-a}}{a+1}$

$\frac{\sqrt{1-a}}{1+a}\ge \frac{3(1-a)\sqrt{6}}{8}\Leftrightarrow (1-a)(3a+1)^2(3a+5)\ge 0$

$\Rightarrow A\ge \frac{3\sqrt{6}}{2}$

Đẳng thức xảy ra khi x=y=z=3.Vậy GTNN của A là $\frac{3\sqrt{6}}{2}$.



#209
daicahuyvn

daicahuyvn

    Hạ sĩ

  • Thành viên
  • 93 Bài viết
 

Cho $a,b,c>0$ thỏa $\frac{1}{a+1}+\frac{1}{b+1}+\frac{1}{c+1}=2$. Tìm GTNN của $$P=\frac{3}{8a^2+1}+\frac{32(b^2+c^2)+10}{64b^2c^2+16bc+1}$$

 

$\frac{1}{1+a}<1\Rightarrow \frac{1}{1+b}+\frac{1}{1+c}\ge 1 \Rightarrow bc\le 1$

$2=\frac{1}{1+a}+\frac{1}{1+b}+\frac{1}{1+c}\le \frac{1}{1+a}+\frac{2}{1+\sqrt{bc}}\Rightarrow a\le \frac{1}{2\sqrt{bc}}-\frac{1}{2}$

Đặt $\sqrt{bc}=x$.$b^2+c^2\ge 2bc=2x^2$

$P\ge \frac{3x^2}{3x^2-4x+2}+\frac{64x^2+10}{(8x^2+1)^2}=f(x)$

f(x) nhỏ nhất khi $x\approx 0,581196$ là nghiệm của 1 pt bậc 7.

$\min P \approx 3,778$


Bài viết đã được chỉnh sửa nội dung bởi daicahuyvn: 05-06-2014 - 06:57


#210
25 minutes

25 minutes

    Thành viên nổi bật 2015

  • Hiệp sỹ
  • 2795 Bài viết

Cho $x,y,z\in (0,1)$ thoã điều kiện xy+yz+zx=1

Tìm giá trị nhỏ nhất của hàm số: $P=\frac{x}{1-x^2}+\frac{y}{1-y^2}+\frac{z}{1-z^2}$

Chứng minh 

     $\sum \frac{x}{1-x^2}\geqslant \sum \frac{3x^2\sqrt{3}}{2}\geqslant \frac{3\sqrt{3}}{2}(xy+yz+zx)=\frac{3\sqrt{3}}{2} $


Hãy theo đuổi đam mê, thành công sẽ theo đuổi bạn.



Thảo luận BĐT ôn thi Đại học tại đây


#211
taovanchi

taovanchi

    Binh nhì

  • Thành viên
  • 14 Bài viết

Cho các số thực không âm thoả mãn $z\geq y\geq x$, xy+yz+zx> 0. Tìm giá trị nhỏ nhất của biểu thức 

$2\sqrt{\frac{x}{y+z}}+2\sqrt{\frac{y}{x+z}}+3\sqrt[3]{\frac{z}{x+y}}$.

 

 

Bài này cầng chứng minh bổ đề $\sqrt{\frac{x}{y+z}}+\sqrt{\frac{y}{x+z}}\geq \sqrt{\frac{x+y}{z}}$ nhưng mình không biết xuất phát từ đâu tìm ra bổ đề đó. Mong mọi người giúp !!!1

sử dụng BDT cosi: $\sqrt{x(y+z)}\leq \frac{x+y+z}{2}\Rightarrow 1\geq 2\frac{\sqrt{x(y+z)}}{x+y+z}$

nhân cả 2 vế cho: $\sqrt{\frac{x}{y+z}}\Rightarrow \sqrt{\frac{x}{y+z}}\geq \frac{2x}{x+y+z}$

tương tự trên: $\sqrt{\frac{x}{y+z}}+\sqrt{\frac{y}{x+z}}\geq \frac{2(x+y)}{(x+y)+z}\geq \frac{2(x+y)}{2\sqrt{(x+y)z}}\doteq \sqrt{\frac{x+y}{z}}$



#212
NDP

NDP

    Hạ sĩ

  • Thành viên
  • 74 Bài viết

ý tớ là tại sao có được cái bất đẳng thức đầu tiên đó. Sao bạn có được điều đó. Chứ không phải chứng minh nó. Tại sao $\frac{x}{1-x^2}\geqslant \frac{3\sqrt{3}x^2}{2}$ mà không phải là  $\frac{x}{1-x^2}\geqslant \frac{x^2}{2}$

Dựa vào tính chất này nè bạn (Cái này là thầy Ngọc viết)

Ta biết rằng nếu hai đò thị y=f(x) và y=g(x) tiếp xúc nhau tại x$_{o}$ thì tồn tại một khoảng $(\alpha ;\beta )$ chứa x$_{o}$ .sao cho trên khoảng đó, đò thị này nằm dưới đồ thị kia, tức là

                  $f(x)\geq g(x),\forall x\varepsilon (\alpha ;\beta )$

         hoăc    $f(x)\leq g(x),\forall x\varepsilon (\alpha ;\beta )$

Sử dụng tính chất này , ta có thể chứng minh một số bất đẳng thức dạng như sau :

Cho các số thực $a_{1},a_{2},a_{3},...,a_{n}\varepsilon D$ thoả mãn

$g(a_{1})+g(a_{2})+...+g(a_{n})\geq (\leq )ng(m)$, Với m$\varepsilon D$.Chứng minh rằng:

          $f(a_{1})+f(a_{2})+...+f(a_{n})\geq (\leq )nf(m)$

Để giải bài toán này , ta cần tìm các số thực a ,b sao cho đồ thị hàm số y=f(x) tiếp xúc với đò thị hàm số y=ag(x)+b tại x$_{o}$=m

Tức là hệ phương trình:

$\begin{cases} & \text{ } f(x)=ag(x)+b \\ & \text{ } f^{'}(x)=ag^{'}(x) \end{cases}$ có một nghiệm x$_{o}$=m.

Dựa vào điều này ta tìm được a,b.Sau đó chứng minh đò thị này nằm dưới hoặc nằm trên đồ thị kia khoảng $(\alpha ;\beta )$ nào đó có thể

Đôi khi, ta chỉ cần đồ thị hàm số y=f(x) nằm dưới (hhoặ nằm trên) đồ thị hàm số y=ag(x)+b trên khoảng $(\alpha ;\beta )$ (haặc đoạn $[\alpha ;\beta ]$) cần thiết, mà không cần tiếp xúc.

+)Quay lại vơí bài toán

Ta dự đoán dấu '=' xẩy ra tại tâm ta sẽ tìm a,b sao cho f(x)=$\frac{x}{1-x^{2}}\geq ax^{2}+b=g(x)$

Hại đồ thị này tiếp xúc nhau tại $x_{o}=\frac{1}{\sqrt{3}}$ khi

    $\begin{cases} & \text{ } \frac{x}{1-x^{2}}=ax^{2}+b \\ & \text{ } \frac{x^{2}+1}{(1-x^{2})^{2}}=2ax \end{cases}$ (Khi x=$\frac{1}{\sqrt{3}})$

Giải hệ này ta tìm được a=$\frac{3\sqrt{3}}{2}$,b=0


 $\sqrt{O}$ve math

 

:ukliam2:  :ukliam2:  :ukliam2:

 

 

:ukliam2: Learn from yesterday, live for today, hope for tomorrow and the important thing is not to stop questioning :like

                                       

                         my facebook: https://www.facebook.com/NDPA1K46
           my email: [email protected]
 
 
 

 


#213
25 minutes

25 minutes

    Thành viên nổi bật 2015

  • Hiệp sỹ
  • 2795 Bài viết

Cho a,b,c>0
Tìm GTNN của: P=$\frac{4a^3 + 3b^2 + 2c^3 - 3b^2c}{(a + b + c)^3}$

Ta có $P=\frac{4a^3+b^3+c^3+(2b^3+c^3-3b^2c)}{(a+b+c)}^3\geqslant \frac{4a^3+b^3+c^3}{(a+b+c)^3}$

Áp dụng bất đẳng thức Holder ta có 

 $(4a^3+b^3+c^3)(\frac{1}{2}+1+1)(\frac{1}{2}+1+1)\geqslant (a+b+c)^3$

$\Rightarrow P\geqslant \frac{4}{25}$

Đẳng thức xảy ra khi...


Hãy theo đuổi đam mê, thành công sẽ theo đuổi bạn.



Thảo luận BĐT ôn thi Đại học tại đây


#214
lahantaithe99

lahantaithe99

    Trung úy

  • Thành viên
  • 883 Bài viết

Tìm giá trị lớn nhất của biểu thức

$$F=\dfrac{a}{\sqrt{a^2+b+c}}+\dfrac{b}{\sqrt{b^2+c+a}}+\dfrac{c}{\sqrt{c^2+c+a}}$$

Trong đó $a,b,c$ là các số thực dương thỏa mãn$a^2+b^2+c^2=3$

Áp dụng Bunhia

 

$(a^2+b+c)(1+b+c)\geqslant (a+b+c)^2\Rightarrow \sqrt{a^2+b+c}\geq \frac{a+b+c}{\sqrt{1+b+c}}$

 

Suy ra $F\leqslant \frac{\sum a\sqrt{1+b+c}}{a+b+c}\leqslant \frac{\sqrt{(a+b+c)(a+b+c+2ab+2bc+2ca)}}{a+b+c}=\frac{\sqrt{3(3+2a+2b+2c)}}{a+b+c}$

 

$\Leftrightarrow F\leqslant \sqrt{\frac{a+b+c+2ab+2bc+2ac}{a+b+c}}$

 

Lại có $2ab+2bc+2ac\leqslant \frac{2(a+b+c)^2}{3}\Rightarrow F\leqslant \sqrt{\frac{3+2(a+b+c)}{3}}\leqslant \sqrt{3}$



#215
hoangmac

hoangmac

    Binh nhì

  • Thành viên
  • 10 Bài viết

Tìm giá trị nhỏ nhất của biểu thức

$$P=(xy+yz+2zx)^2-\dfrac{8}{(x+y+z)^2-xy-yz+2}$$

Trong đó $x,y,z $là các số thực thỏa mãn điều kiện  $x^2+y^2+z^2=1$

 

Đặt $t=xy+yz+2zx = (x+y+z)^2-y(x+z)-1\geq (x+y+z)^2 -\dfrac{(x+y+z)^2}{4} -1 \geq -1$

Khi đó: $P=t^2-\dfrac{8}{t+3}=f(t)$

$f(t)'= \dfrac{2(t+1)^2(t+4)}{(t+3)^2} \geq 0$

Nên $f(t) \geq f(-1)=-3$

Vậy $Pmin=-3$



#216
25 minutes

25 minutes

    Thành viên nổi bật 2015

  • Hiệp sỹ
  • 2795 Bài viết

Cho các số thực x, y thoả mãn: $x-3\sqrt{x+1}=3\sqrt{y+2}-y$. 

Tìm GTLN, GTNN của $P=x+y$

ĐK $x \geqslant -1$, $y \geqslant -2$

Từ giả thiết ta có $x+y=3\sqrt{x+1}+3\sqrt{y+2}\leqslant 6\sqrt{\frac{x+y+3}{2}}$

           $\Rightarrow x+y\leqslant \frac{18+6\sqrt{15}}{2}$

Đẳng thức xảy ra khi $\left\{\begin{matrix} x+1=y+2\\x+y=\frac{18+6\sqrt{15}}{2} \end{matrix}\right.$

Lại có $(x+y)^2=9(x+y+3)+18\sqrt{(x+1)(y+2)}\geqslant 9(x+y+3)$

         $\Rightarrow x+y\geqslant \frac{9+3\sqrt{21}}{2}$

Đẳng thức xảy ra khi $\left\{\begin{matrix} x+y=\frac{9+3\sqrt{21}}{2}\\(x+1)(y+2)=0 \end{matrix}\right.$


Hãy theo đuổi đam mê, thành công sẽ theo đuổi bạn.



Thảo luận BĐT ôn thi Đại học tại đây


#217
NDP

NDP

    Hạ sĩ

  • Thành viên
  • 74 Bài viết

Bài ??? : Cho $a,b,c$ là các số thực không đồng thời bằng $0$ thỏa mãn $a^2+b^2+c^2=2(ab+bc+ca)$

Tìm GTNN và GTLN của $P=\frac{a^3+b^3+c^3}{(a+b+c)(a^2+b^2+c^2)}$

Lời giải 

Viết giả thiết lại thành $(x+y+z)^{2}=2x^{2}+2z^{2}+2y^{2}$

Nên P=2$[(\frac{x}{x+y+z})^{3}+(\frac{y}{x+y+z})^{3}+(\frac{z}{x+y+z})^{3}]$

Đặt $a=\frac{x}{x+y+z},b=\frac{y}{x+y+z},c=\frac{z}{x+y+z}$

     $\Rightarrow \begin{cases} & \text{ } a+b+c=1 \\ & \text{ } ab+bc+ca=\frac{1}{4} \end{cases}$

Lúc đó P=$2[a^{3}+(b+c)^{3}-2bc(b+c)]$

 Ta co : $\begin{cases} & \text{ } b+c=1-a \\ & \text{ } bc=\frac{1}{4}-a(1-a) \end{cases}$

 $\Rightarrow P=2[a^{3}+(1-b)^{3}+3(a-1)(a^{2}-a+\frac{1}{4})]$

                   =$2(3a^{3}-3a^{2}+\frac{3}{4}a+\frac{1}{4})$

Ta co $\frac{1}{4}=a(1-a)+bc\leq a(1-a)+\frac{(1-a)^{2}}{4}$ $\Rightarrow a\varepsilon [0,\frac{2}{3}]$

Xet f(a)=$3a^{3}-3a^{2}+\frac{3}{4}a+\frac{1}{4}$ tren $a\varepsilon [0,\frac{2}{3}]$

Ta duoc Pmin=$\frac{1}{2}$ khi a=o,b=c=$\frac{1}{2}$ hoac a=$\frac{1}{2}$ ,b=c=$\frac{1}{4}$ va cac hoan vi tuong ung

          Pmax=$\frac{11}{18}$ khi ..........


 $\sqrt{O}$ve math

 

:ukliam2:  :ukliam2:  :ukliam2:

 

 

:ukliam2: Learn from yesterday, live for today, hope for tomorrow and the important thing is not to stop questioning :like

                                       

                         my facebook: https://www.facebook.com/NDPA1K46
           my email: [email protected]
 
 
 

 


#218
anhxuanfarastar

anhxuanfarastar

    Sĩ quan

  • Thành viên
  • 368 Bài viết

Bài ??? : Cho $a,b,c$ là các số thực không đồng thời bằng $0$ thỏa mãn $a^2+b^2+c^2=2(ab+bc+ca)$

Tìm GTNN và GTLN của $P=\frac{a^3+b^3+c^3}{(a+b+c)(a^2+b^2+c^2)}$

Từ giả thiết suy ra $(a+b+c)^{2}=2(a^2+b^2+c^2)\neq 0$

Đặt: $x=\frac{4a}{a+b+c}; y=\frac{4b}{a+b+c}; z=\frac{4c}{a+b+c}$

* Khi đó có $$\left\{\begin{matrix} & x+y+z=4 & \\ & xy+yz+zx=4 & \end{matrix}\right. \Leftrightarrow \left\{\begin{matrix} & y+z=4-x & \\ & ya=x^2-4x+4 & \end{matrix}\right. \Rightarrow (4-x)^{2}\geq 4(x^2-4x+4)\Leftrightarrow 0\leq x\leq \frac{8}{3}$$

 

Suy ra $P=\frac{2(a^3+b^3+c^3)}{(a+b+c)^{3}}=\frac{1}{32}(3x^3-12x^2+12z+16), x\in [0;\frac{8}{3}]$

Xét hàm trên tìm được $minP=P(0)=0,5  ;   maxP=P(\frac{2}{3})=P(\frac{8}{3})=\frac{11}{18}$


Bài viết đã được chỉnh sửa nội dung bởi anhxuanfarastar: 12-06-2014 - 22:47

INTELLIGENCE IS THE ABILITY TO ADAPT TO CHANGE !!!


#219
NDP

NDP

    Hạ sĩ

  • Thành viên
  • 74 Bài viết

Bài n: Cho $a,b,c >0$ và $\frac{a^2+b^2+c^2}{ab+bc+ca}=\frac{ab+bc+ca+1}{2}$

Tìm GTLN của $P=\sqrt{a^2+b^2+c^2}-\frac{\left | a-b \right |+\left | b-c \right |+\left | c-a \right |}{2}$

                                                              

Lời giải

Không mất tính tổng quát giả sử $a\geq b\geq c$

Lúc đó P=$\sqrt{a^{2}+b^{2}+c^{2}}-(a-c)$

Từ giả thiết và áp dụng $a^{2}+b^{2}+c^{2}\geq ab+bc+ca$ do đó $\frac{ab+bc+ca+1}{2}\geq 1\Rightarrow ab+bc+ca\geq 1$

Đặt t=ab+bc+ca thì $a^{2}+b^{2}+c^{2}=\frac{t^{2}+t}{2}$

Lại có $(a-c)^{2}=(a-b+b-c)^{2}\geq (a-b)^{2}+(b-c)^{2}$

Nên $2(a-c)^{2}=(a-c)^{2}+(b-c)^{2}+(c-a)^{2}=t^{2}-t$

Vậy P$\leq \frac{1}{\sqrt{2}}(\sqrt{t^{2}+t}-\sqrt{t^{2}-t})$

Ta xẽ chúng minh $\sqrt{t^{2}+t}-\sqrt{t^{2}-t}\leq \sqrt{2}$

                     $\Leftrightarrow \sqrt{t^{2}+t}\leq \sqrt{2}+\sqrt{t^{2}-t}$

                     $\Leftrightarrow \sqrt{t-1}(\sqrt{2t}-\sqrt{t-1})\geq 0$(Đúng do t$\geq 1$)

Vậy P$\leq 1$ khi a=b=c=$\frac{1}{\sqrt{3}}$


 $\sqrt{O}$ve math

 

:ukliam2:  :ukliam2:  :ukliam2:

 

 

:ukliam2: Learn from yesterday, live for today, hope for tomorrow and the important thing is not to stop questioning :like

                                       

                         my facebook: https://www.facebook.com/NDPA1K46
           my email: [email protected]
 
 
 

 


#220
NDP

NDP

    Hạ sĩ

  • Thành viên
  • 74 Bài viết

 

Đặt t=ab+bc+ca thì $a^{2}+b^{2}+c^{2}=\frac{t^{2}+t}{2}$

 

Lại có $(a-c)^{2}=(a-b+b-c)^{2}\geq (a-b)^{2}+(b-c)^{2}$

Nên $2(a-c)^{2}=(a-c)^{2}+(b-c)^{2}+(c-a)^{2}=t^{2}-t$

đoạn này mình không hiểu lắm. nhất là chỗ Đặt t=ab+bc+ca thì $a^{2}+b^{2}+c^{2}=\frac{t^{2}+t}{2}$ mình thấy t=ab+bc+ca thì $a^{2}+b^{2}+c^{2}\neq \frac{t^{2}+t}{2}$ 

 

Từ giả thiết như bạn


 $\sqrt{O}$ve math

 

:ukliam2:  :ukliam2:  :ukliam2:

 

 

:ukliam2: Learn from yesterday, live for today, hope for tomorrow and the important thing is not to stop questioning :like

                                       

                         my facebook: https://www.facebook.com/NDPA1K46
           my email: [email protected]
 
 
 

 





1 người đang xem chủ đề

0 thành viên, 1 khách, 0 thành viên ẩn danh